Math, asked by Nehapawar735, 2 months ago

then find matrix B such that AB=I. verify If A= 1 0 0, 5 1 0, 1 3 1
that BA = I.

Answers

Answered by MaheswariS
47

\textbf{Given:}

\mathsf{A=\left(\begin{array}{ccc}1&0&0\\5&1&0\\1&3&1\end{array}\right)}

\textbf{To find:}

\textsf{The matrix B}

\textbf{Solution:}

\textsf{Consider,}

\mathsf{AB=I}

\implies\mathsf{B=A^{-1}}

\mathsf{|A|=\left|\begin{array}{ccc}1&0&0\\5&1&0\\1&3&1\end{array}\right|}

\mathsf{=1(1-0)-0+0=1\neq\,0}

\mathsf{A^{-1}\;exists}

\mathsf{adjA=\left(\begin{array}{ccc}(1-0)&-(5-0)&(15-1)\\-(0-0)&(1-0)&-(3-0)\\(0-0)&-(0-0)&(1-0)\end{array}\right)^T}

\mathsf{adjA=\left(\begin{array}{ccc}1&-5&14\\0&1&-3\\0&0&1\end{array}\right)^T}

\mathsf{adjA=\left(\begin{array}{ccc}1&0&0\\-5&1&0\\14&-3&1\end{array}\right)}

\mathsf{Now,\;A^{-1}=\dfrac{1}{|A|}adjA}

\mathsf{A^{-1}=\left(\begin{array}{ccc}1&0&0\\-5&1&0\\14&-3&1\end{array}\right)}

\implies\boxed{\mathsf{B=\left(\begin{array}{ccc}1&0&0\\-5&1&0\\14&-3&1\end{array}\right)}}

\mathsf{BA}

\mathsf{=\left(\begin{array}{ccc}1&0&0\\-5&1&0\\14&-3&1\end{array}\right)\left(\begin{array}{ccc}1&0&0\\5&1&0\\1&3&1\end{array}\right)}

\mathsf{=\left(\begin{array}{ccc}1+0+0&0+0+0&0+0+0\\-5+5+0&0+1+0&0+0+0\\14-15+1&0-3+3&0+0+1\end{array}\right)}

\mathsf{=\left(\begin{array}{ccc}1&0&0\\0&1&0\\0&0&1\end{array}\right)}

\mathsf{=I}

\mathsf{Hence\;verified}

\textbf{Find more:}

If A is a non singular matrix of order 3,then|adj.A|=​

https://brainly.in/question/19753829

Answered by mahek77777
11

\textbf\red{Given:}

\mathsf{A=\left(\begin{array}{ccc}1&0&0\\5&1&0\\1&3&1\end{array}\right)}

\textbf\red{To find:}

\textsf{The matrix B}

\textbf\red{Solution:}

\textsf{Consider,}

\mathsf{AB=I}

\implies\mathsf{B=A^{-1}}

\mathsf{|A|=\left|\begin{array}{ccc}1&0&0\\5&1&0\\1&3&1\end{array}\right|}

\mathsf{=1(1-0)-0+0=1\neq\,0}

\mathsf{A^{-1}\;exists}

\mathsf{adjA=\left(\begin{array}{ccc}(1-0)&-(5-0)&(15-1)\\-(0-0)&(1-0)&-(3-0)\\(0-0)&-(0-0)&(1-0)\end{array}\right)^T}

\mathsf{adjA=\left(\begin{array}{ccc}1&-5&14\\0&1&-3\\0&0&1\end{array}\right)^T}

\mathsf{adjA=\left(\begin{array}{ccc}1&0&0\\-5&1&0\\14&-3&1\end{array}\right)}

\mathsf{Now,\;A^{-1}=\dfrac{1}{|A|}adjA}

\mathsf{A^{-1}=\left(\begin{array}{ccc}1&0&0\\-5&1&0\\14&-3&1\end{array}\right)}

\implies\boxed{\mathsf{B=\left(\begin{array}{ccc}1&0&0\\-5&1&0\\14&-3&1\end{array}\right)}}

\mathsf{BA}

\mathsf{=\left(\begin{array}{ccc}1&0&0\\-5&1&0\\14&-3&1\end{array}\right)\left(\begin{array}{ccc}1&0&0\\5&1&0\\1&3&1\end{array}\right)}

\mathsf{=\left(\begin{array}{ccc}1+0+0&0+0+0&0+0+0\\-5+5+0&0+1+0&0+0+0\\14-15+1&0-3+3&0+0+1\end{array}\right)}

\mathsf{=\left(\begin{array}{ccc}1&0&0\\0&1&0\\0&0&1\end{array}\right)}

\mathsf{=I}

\mathsf\red{Hence\;verified}

Similar questions